Sum to Infinity of Trigonometry to $pi$












1












$begingroup$


For



$$y=sum_{n=0}^a2cdot2^ncdottanleft(frac{45}{2^n}right)cdotsinleft(frac{90}{2^n}right)^2$$



I am currently working on a proof with a good friend of mine that involves adding more and more triangles to the sides of a regular polygon but keeping the longest diagonal constant until eventually, it becomes a circle. And we ended up with this formula.



4-sided regular→ 8-sided regular→ 16-sided regular→ 32-sided regular→... →n-sided regular



(When n tends to infinity, the area will be equal to that of a circle with the longest diagonal as diameter)



We have already tried Geometric Sequence Infinite Sum, but there does not seem to have a common ratio.



Moreover, we have used our calculator to input the numbers up to 128[(sin ...] and we get the value of 3.140... which is very close to π But we can't be completely sure that the infinity sum really equals to π.



That is why we really need your knowledge of Maths to solve this.



Is there a way to prove that when $a$ tends to infinity, $y$ tends to $pi$?



Thanks in advance and Happy Holidays Everyone! :D










share|cite|improve this question











$endgroup$












  • $begingroup$
    You'll find that simple "Here's the statement of my question, solve it for me" posts will be poorly received. What is better is for you to add context (with an edit): What you understand about the problem, what you've tried so far, etc.; something both to show you are part of the learning experience and to help us guide you to the appropriate help. You can consult this link for further guidance.
    $endgroup$
    – Shaun
    Dec 25 '18 at 13:24






  • 2




    $begingroup$
    Thanks for the tip :D I will edit this right away!
    $endgroup$
    – user629248
    Dec 25 '18 at 13:25






  • 1




    $begingroup$
    Jolly Christmas and Happy Holidays to you, My Good Fellow! :)
    $endgroup$
    – user629248
    Dec 25 '18 at 13:26










  • $begingroup$
    Thank you, and Happy Holidays to you too! Let me know once you've edited the question, then I'll probably upvote :)
    $endgroup$
    – Shaun
    Dec 25 '18 at 13:32






  • 1




    $begingroup$
    Hey by the way, Shaun, can you tell me how can I type Equations Symbols when I submit my Question?
    $endgroup$
    – user629248
    Dec 25 '18 at 13:43
















1












$begingroup$


For



$$y=sum_{n=0}^a2cdot2^ncdottanleft(frac{45}{2^n}right)cdotsinleft(frac{90}{2^n}right)^2$$



I am currently working on a proof with a good friend of mine that involves adding more and more triangles to the sides of a regular polygon but keeping the longest diagonal constant until eventually, it becomes a circle. And we ended up with this formula.



4-sided regular→ 8-sided regular→ 16-sided regular→ 32-sided regular→... →n-sided regular



(When n tends to infinity, the area will be equal to that of a circle with the longest diagonal as diameter)



We have already tried Geometric Sequence Infinite Sum, but there does not seem to have a common ratio.



Moreover, we have used our calculator to input the numbers up to 128[(sin ...] and we get the value of 3.140... which is very close to π But we can't be completely sure that the infinity sum really equals to π.



That is why we really need your knowledge of Maths to solve this.



Is there a way to prove that when $a$ tends to infinity, $y$ tends to $pi$?



Thanks in advance and Happy Holidays Everyone! :D










share|cite|improve this question











$endgroup$












  • $begingroup$
    You'll find that simple "Here's the statement of my question, solve it for me" posts will be poorly received. What is better is for you to add context (with an edit): What you understand about the problem, what you've tried so far, etc.; something both to show you are part of the learning experience and to help us guide you to the appropriate help. You can consult this link for further guidance.
    $endgroup$
    – Shaun
    Dec 25 '18 at 13:24






  • 2




    $begingroup$
    Thanks for the tip :D I will edit this right away!
    $endgroup$
    – user629248
    Dec 25 '18 at 13:25






  • 1




    $begingroup$
    Jolly Christmas and Happy Holidays to you, My Good Fellow! :)
    $endgroup$
    – user629248
    Dec 25 '18 at 13:26










  • $begingroup$
    Thank you, and Happy Holidays to you too! Let me know once you've edited the question, then I'll probably upvote :)
    $endgroup$
    – Shaun
    Dec 25 '18 at 13:32






  • 1




    $begingroup$
    Hey by the way, Shaun, can you tell me how can I type Equations Symbols when I submit my Question?
    $endgroup$
    – user629248
    Dec 25 '18 at 13:43














1












1








1





$begingroup$


For



$$y=sum_{n=0}^a2cdot2^ncdottanleft(frac{45}{2^n}right)cdotsinleft(frac{90}{2^n}right)^2$$



I am currently working on a proof with a good friend of mine that involves adding more and more triangles to the sides of a regular polygon but keeping the longest diagonal constant until eventually, it becomes a circle. And we ended up with this formula.



4-sided regular→ 8-sided regular→ 16-sided regular→ 32-sided regular→... →n-sided regular



(When n tends to infinity, the area will be equal to that of a circle with the longest diagonal as diameter)



We have already tried Geometric Sequence Infinite Sum, but there does not seem to have a common ratio.



Moreover, we have used our calculator to input the numbers up to 128[(sin ...] and we get the value of 3.140... which is very close to π But we can't be completely sure that the infinity sum really equals to π.



That is why we really need your knowledge of Maths to solve this.



Is there a way to prove that when $a$ tends to infinity, $y$ tends to $pi$?



Thanks in advance and Happy Holidays Everyone! :D










share|cite|improve this question











$endgroup$




For



$$y=sum_{n=0}^a2cdot2^ncdottanleft(frac{45}{2^n}right)cdotsinleft(frac{90}{2^n}right)^2$$



I am currently working on a proof with a good friend of mine that involves adding more and more triangles to the sides of a regular polygon but keeping the longest diagonal constant until eventually, it becomes a circle. And we ended up with this formula.



4-sided regular→ 8-sided regular→ 16-sided regular→ 32-sided regular→... →n-sided regular



(When n tends to infinity, the area will be equal to that of a circle with the longest diagonal as diameter)



We have already tried Geometric Sequence Infinite Sum, but there does not seem to have a common ratio.



Moreover, we have used our calculator to input the numbers up to 128[(sin ...] and we get the value of 3.140... which is very close to π But we can't be completely sure that the infinity sum really equals to π.



That is why we really need your knowledge of Maths to solve this.



Is there a way to prove that when $a$ tends to infinity, $y$ tends to $pi$?



Thanks in advance and Happy Holidays Everyone! :D







summation infinity pi






share|cite|improve this question















share|cite|improve this question













share|cite|improve this question




share|cite|improve this question








edited Dec 25 '18 at 13:45

























asked Dec 25 '18 at 13:17







user629248



















  • $begingroup$
    You'll find that simple "Here's the statement of my question, solve it for me" posts will be poorly received. What is better is for you to add context (with an edit): What you understand about the problem, what you've tried so far, etc.; something both to show you are part of the learning experience and to help us guide you to the appropriate help. You can consult this link for further guidance.
    $endgroup$
    – Shaun
    Dec 25 '18 at 13:24






  • 2




    $begingroup$
    Thanks for the tip :D I will edit this right away!
    $endgroup$
    – user629248
    Dec 25 '18 at 13:25






  • 1




    $begingroup$
    Jolly Christmas and Happy Holidays to you, My Good Fellow! :)
    $endgroup$
    – user629248
    Dec 25 '18 at 13:26










  • $begingroup$
    Thank you, and Happy Holidays to you too! Let me know once you've edited the question, then I'll probably upvote :)
    $endgroup$
    – Shaun
    Dec 25 '18 at 13:32






  • 1




    $begingroup$
    Hey by the way, Shaun, can you tell me how can I type Equations Symbols when I submit my Question?
    $endgroup$
    – user629248
    Dec 25 '18 at 13:43


















  • $begingroup$
    You'll find that simple "Here's the statement of my question, solve it for me" posts will be poorly received. What is better is for you to add context (with an edit): What you understand about the problem, what you've tried so far, etc.; something both to show you are part of the learning experience and to help us guide you to the appropriate help. You can consult this link for further guidance.
    $endgroup$
    – Shaun
    Dec 25 '18 at 13:24






  • 2




    $begingroup$
    Thanks for the tip :D I will edit this right away!
    $endgroup$
    – user629248
    Dec 25 '18 at 13:25






  • 1




    $begingroup$
    Jolly Christmas and Happy Holidays to you, My Good Fellow! :)
    $endgroup$
    – user629248
    Dec 25 '18 at 13:26










  • $begingroup$
    Thank you, and Happy Holidays to you too! Let me know once you've edited the question, then I'll probably upvote :)
    $endgroup$
    – Shaun
    Dec 25 '18 at 13:32






  • 1




    $begingroup$
    Hey by the way, Shaun, can you tell me how can I type Equations Symbols when I submit my Question?
    $endgroup$
    – user629248
    Dec 25 '18 at 13:43
















$begingroup$
You'll find that simple "Here's the statement of my question, solve it for me" posts will be poorly received. What is better is for you to add context (with an edit): What you understand about the problem, what you've tried so far, etc.; something both to show you are part of the learning experience and to help us guide you to the appropriate help. You can consult this link for further guidance.
$endgroup$
– Shaun
Dec 25 '18 at 13:24




$begingroup$
You'll find that simple "Here's the statement of my question, solve it for me" posts will be poorly received. What is better is for you to add context (with an edit): What you understand about the problem, what you've tried so far, etc.; something both to show you are part of the learning experience and to help us guide you to the appropriate help. You can consult this link for further guidance.
$endgroup$
– Shaun
Dec 25 '18 at 13:24




2




2




$begingroup$
Thanks for the tip :D I will edit this right away!
$endgroup$
– user629248
Dec 25 '18 at 13:25




$begingroup$
Thanks for the tip :D I will edit this right away!
$endgroup$
– user629248
Dec 25 '18 at 13:25




1




1




$begingroup$
Jolly Christmas and Happy Holidays to you, My Good Fellow! :)
$endgroup$
– user629248
Dec 25 '18 at 13:26




$begingroup$
Jolly Christmas and Happy Holidays to you, My Good Fellow! :)
$endgroup$
– user629248
Dec 25 '18 at 13:26












$begingroup$
Thank you, and Happy Holidays to you too! Let me know once you've edited the question, then I'll probably upvote :)
$endgroup$
– Shaun
Dec 25 '18 at 13:32




$begingroup$
Thank you, and Happy Holidays to you too! Let me know once you've edited the question, then I'll probably upvote :)
$endgroup$
– Shaun
Dec 25 '18 at 13:32




1




1




$begingroup$
Hey by the way, Shaun, can you tell me how can I type Equations Symbols when I submit my Question?
$endgroup$
– user629248
Dec 25 '18 at 13:43




$begingroup$
Hey by the way, Shaun, can you tell me how can I type Equations Symbols when I submit my Question?
$endgroup$
– user629248
Dec 25 '18 at 13:43










2 Answers
2






active

oldest

votes


















2












$begingroup$

Hint:



$$2tan Acdotsin^22A=2(4sin^3Acos A)=(3sin A-sin3A)2cos A$$
$$=3sin2A-(sin4A+sin2A)=2sin2A-sin4A$$



which clearly shows Telescopic form






share|cite|improve this answer









$endgroup$





















    0












    $begingroup$

    When I come across things like these: (if you do not need to prove it manually)
    Wolfram Alpha computation






    share|cite|improve this answer









    $endgroup$













    • $begingroup$
      Thank you! I have heard of Wolfram before, but is there a manually proof for this question?
      $endgroup$
      – user629248
      Dec 25 '18 at 14:02










    • $begingroup$
      Easy to fool yourself here. If the sum happened to be $pi + 10^{-16}$ then WA would still say True to your query. Just try this (it's $pi$ up to 15 digits). In this case it seems it evaluates both sides numerically (to floating point accuracy) and checks if they agree. But it doesn't tell you that this is what it does.
      $endgroup$
      – Winther
      Dec 25 '18 at 14:43












    • $begingroup$
      To check this try to ask WA to just evaluate the sum, it doesn't evaluate it to $pi$ symbolically.
      $endgroup$
      – Winther
      Dec 25 '18 at 14:44











    Your Answer





    StackExchange.ifUsing("editor", function () {
    return StackExchange.using("mathjaxEditing", function () {
    StackExchange.MarkdownEditor.creationCallbacks.add(function (editor, postfix) {
    StackExchange.mathjaxEditing.prepareWmdForMathJax(editor, postfix, [["$", "$"], ["\\(","\\)"]]);
    });
    });
    }, "mathjax-editing");

    StackExchange.ready(function() {
    var channelOptions = {
    tags: "".split(" "),
    id: "69"
    };
    initTagRenderer("".split(" "), "".split(" "), channelOptions);

    StackExchange.using("externalEditor", function() {
    // Have to fire editor after snippets, if snippets enabled
    if (StackExchange.settings.snippets.snippetsEnabled) {
    StackExchange.using("snippets", function() {
    createEditor();
    });
    }
    else {
    createEditor();
    }
    });

    function createEditor() {
    StackExchange.prepareEditor({
    heartbeatType: 'answer',
    autoActivateHeartbeat: false,
    convertImagesToLinks: true,
    noModals: true,
    showLowRepImageUploadWarning: true,
    reputationToPostImages: 10,
    bindNavPrevention: true,
    postfix: "",
    imageUploader: {
    brandingHtml: "Powered by u003ca class="icon-imgur-white" href="https://imgur.com/"u003eu003c/au003e",
    contentPolicyHtml: "User contributions licensed under u003ca href="https://creativecommons.org/licenses/by-sa/3.0/"u003ecc by-sa 3.0 with attribution requiredu003c/au003e u003ca href="https://stackoverflow.com/legal/content-policy"u003e(content policy)u003c/au003e",
    allowUrls: true
    },
    noCode: true, onDemand: true,
    discardSelector: ".discard-answer"
    ,immediatelyShowMarkdownHelp:true
    });


    }
    });














    draft saved

    draft discarded


















    StackExchange.ready(
    function () {
    StackExchange.openid.initPostLogin('.new-post-login', 'https%3a%2f%2fmath.stackexchange.com%2fquestions%2f3052096%2fsum-to-infinity-of-trigonometry-to-pi%23new-answer', 'question_page');
    }
    );

    Post as a guest















    Required, but never shown
























    2 Answers
    2






    active

    oldest

    votes








    2 Answers
    2






    active

    oldest

    votes









    active

    oldest

    votes






    active

    oldest

    votes









    2












    $begingroup$

    Hint:



    $$2tan Acdotsin^22A=2(4sin^3Acos A)=(3sin A-sin3A)2cos A$$
    $$=3sin2A-(sin4A+sin2A)=2sin2A-sin4A$$



    which clearly shows Telescopic form






    share|cite|improve this answer









    $endgroup$


















      2












      $begingroup$

      Hint:



      $$2tan Acdotsin^22A=2(4sin^3Acos A)=(3sin A-sin3A)2cos A$$
      $$=3sin2A-(sin4A+sin2A)=2sin2A-sin4A$$



      which clearly shows Telescopic form






      share|cite|improve this answer









      $endgroup$
















        2












        2








        2





        $begingroup$

        Hint:



        $$2tan Acdotsin^22A=2(4sin^3Acos A)=(3sin A-sin3A)2cos A$$
        $$=3sin2A-(sin4A+sin2A)=2sin2A-sin4A$$



        which clearly shows Telescopic form






        share|cite|improve this answer









        $endgroup$



        Hint:



        $$2tan Acdotsin^22A=2(4sin^3Acos A)=(3sin A-sin3A)2cos A$$
        $$=3sin2A-(sin4A+sin2A)=2sin2A-sin4A$$



        which clearly shows Telescopic form







        share|cite|improve this answer












        share|cite|improve this answer



        share|cite|improve this answer










        answered Dec 25 '18 at 13:26









        lab bhattacharjeelab bhattacharjee

        226k15158275




        226k15158275























            0












            $begingroup$

            When I come across things like these: (if you do not need to prove it manually)
            Wolfram Alpha computation






            share|cite|improve this answer









            $endgroup$













            • $begingroup$
              Thank you! I have heard of Wolfram before, but is there a manually proof for this question?
              $endgroup$
              – user629248
              Dec 25 '18 at 14:02










            • $begingroup$
              Easy to fool yourself here. If the sum happened to be $pi + 10^{-16}$ then WA would still say True to your query. Just try this (it's $pi$ up to 15 digits). In this case it seems it evaluates both sides numerically (to floating point accuracy) and checks if they agree. But it doesn't tell you that this is what it does.
              $endgroup$
              – Winther
              Dec 25 '18 at 14:43












            • $begingroup$
              To check this try to ask WA to just evaluate the sum, it doesn't evaluate it to $pi$ symbolically.
              $endgroup$
              – Winther
              Dec 25 '18 at 14:44
















            0












            $begingroup$

            When I come across things like these: (if you do not need to prove it manually)
            Wolfram Alpha computation






            share|cite|improve this answer









            $endgroup$













            • $begingroup$
              Thank you! I have heard of Wolfram before, but is there a manually proof for this question?
              $endgroup$
              – user629248
              Dec 25 '18 at 14:02










            • $begingroup$
              Easy to fool yourself here. If the sum happened to be $pi + 10^{-16}$ then WA would still say True to your query. Just try this (it's $pi$ up to 15 digits). In this case it seems it evaluates both sides numerically (to floating point accuracy) and checks if they agree. But it doesn't tell you that this is what it does.
              $endgroup$
              – Winther
              Dec 25 '18 at 14:43












            • $begingroup$
              To check this try to ask WA to just evaluate the sum, it doesn't evaluate it to $pi$ symbolically.
              $endgroup$
              – Winther
              Dec 25 '18 at 14:44














            0












            0








            0





            $begingroup$

            When I come across things like these: (if you do not need to prove it manually)
            Wolfram Alpha computation






            share|cite|improve this answer









            $endgroup$



            When I come across things like these: (if you do not need to prove it manually)
            Wolfram Alpha computation







            share|cite|improve this answer












            share|cite|improve this answer



            share|cite|improve this answer










            answered Dec 25 '18 at 13:57









            yigoliyigoli

            177




            177












            • $begingroup$
              Thank you! I have heard of Wolfram before, but is there a manually proof for this question?
              $endgroup$
              – user629248
              Dec 25 '18 at 14:02










            • $begingroup$
              Easy to fool yourself here. If the sum happened to be $pi + 10^{-16}$ then WA would still say True to your query. Just try this (it's $pi$ up to 15 digits). In this case it seems it evaluates both sides numerically (to floating point accuracy) and checks if they agree. But it doesn't tell you that this is what it does.
              $endgroup$
              – Winther
              Dec 25 '18 at 14:43












            • $begingroup$
              To check this try to ask WA to just evaluate the sum, it doesn't evaluate it to $pi$ symbolically.
              $endgroup$
              – Winther
              Dec 25 '18 at 14:44


















            • $begingroup$
              Thank you! I have heard of Wolfram before, but is there a manually proof for this question?
              $endgroup$
              – user629248
              Dec 25 '18 at 14:02










            • $begingroup$
              Easy to fool yourself here. If the sum happened to be $pi + 10^{-16}$ then WA would still say True to your query. Just try this (it's $pi$ up to 15 digits). In this case it seems it evaluates both sides numerically (to floating point accuracy) and checks if they agree. But it doesn't tell you that this is what it does.
              $endgroup$
              – Winther
              Dec 25 '18 at 14:43












            • $begingroup$
              To check this try to ask WA to just evaluate the sum, it doesn't evaluate it to $pi$ symbolically.
              $endgroup$
              – Winther
              Dec 25 '18 at 14:44
















            $begingroup$
            Thank you! I have heard of Wolfram before, but is there a manually proof for this question?
            $endgroup$
            – user629248
            Dec 25 '18 at 14:02




            $begingroup$
            Thank you! I have heard of Wolfram before, but is there a manually proof for this question?
            $endgroup$
            – user629248
            Dec 25 '18 at 14:02












            $begingroup$
            Easy to fool yourself here. If the sum happened to be $pi + 10^{-16}$ then WA would still say True to your query. Just try this (it's $pi$ up to 15 digits). In this case it seems it evaluates both sides numerically (to floating point accuracy) and checks if they agree. But it doesn't tell you that this is what it does.
            $endgroup$
            – Winther
            Dec 25 '18 at 14:43






            $begingroup$
            Easy to fool yourself here. If the sum happened to be $pi + 10^{-16}$ then WA would still say True to your query. Just try this (it's $pi$ up to 15 digits). In this case it seems it evaluates both sides numerically (to floating point accuracy) and checks if they agree. But it doesn't tell you that this is what it does.
            $endgroup$
            – Winther
            Dec 25 '18 at 14:43














            $begingroup$
            To check this try to ask WA to just evaluate the sum, it doesn't evaluate it to $pi$ symbolically.
            $endgroup$
            – Winther
            Dec 25 '18 at 14:44




            $begingroup$
            To check this try to ask WA to just evaluate the sum, it doesn't evaluate it to $pi$ symbolically.
            $endgroup$
            – Winther
            Dec 25 '18 at 14:44


















            draft saved

            draft discarded




















































            Thanks for contributing an answer to Mathematics Stack Exchange!


            • Please be sure to answer the question. Provide details and share your research!

            But avoid



            • Asking for help, clarification, or responding to other answers.

            • Making statements based on opinion; back them up with references or personal experience.


            Use MathJax to format equations. MathJax reference.


            To learn more, see our tips on writing great answers.




            draft saved


            draft discarded














            StackExchange.ready(
            function () {
            StackExchange.openid.initPostLogin('.new-post-login', 'https%3a%2f%2fmath.stackexchange.com%2fquestions%2f3052096%2fsum-to-infinity-of-trigonometry-to-pi%23new-answer', 'question_page');
            }
            );

            Post as a guest















            Required, but never shown





















































            Required, but never shown














            Required, but never shown












            Required, but never shown







            Required, but never shown

































            Required, but never shown














            Required, but never shown












            Required, but never shown







            Required, but never shown







            Popular posts from this blog

            To store a contact into the json file from server.js file using a class in NodeJS

            Redirect URL with Chrome Remote Debugging Android Devices

            Dieringhausen